Difference between revisions of "1989 AIME Problems/Problem 1"

m
Line 3: Line 3:
  
 
== Solution ==
 
== Solution ==
{{solution}}
+
Let's call our four consecutive integers <math>(n-1), n, (n+1), (n+2)</math>.  Notice that <math>(n-1)(n)(n+1)(n+2)+1=(n^2+n)^2-2(n^2+n)+1 \Rightarrow (n^2+n-1)^2</math>.  Thus, <math>\sqrt{(31)(30)(29)(28)+1} = (29^2+29-1) = 869</math>
  
 
== See also ==
 
== See also ==
 
* [[1989 AIME Problems/Problem 2|Next Problem]]
 
* [[1989 AIME Problems/Problem 2|Next Problem]]
 
* [[1989 AIME Problems]]
 
* [[1989 AIME Problems]]

Revision as of 22:32, 25 February 2007

Problem

Compute $\sqrt{(31)(30)(29)(28)+1}$.

Solution

Let's call our four consecutive integers $(n-1), n, (n+1), (n+2)$. Notice that $(n-1)(n)(n+1)(n+2)+1=(n^2+n)^2-2(n^2+n)+1 \Rightarrow (n^2+n-1)^2$. Thus, $\sqrt{(31)(30)(29)(28)+1} = (29^2+29-1) = 869$

See also